常春藤专注小托福、SSAT、AP、IB、A-Level等最新考试时间、考试资讯。

您当前位置: 首页 > LSAT专区 > LSAT备考 > LSAT推理
  • LSAT分析推理之官网样题(五)

    2023-04-07 11:54:52 来源:网络
    文字:

    常春藤LSAT频道在此与大家分享LSAC官网上公布的LSAT分析推理样题(五),希望对大家备考LSAT分析推理有所帮助。

    Passage for Questions 7 and 8

      On a particular Saturday, a student will perform six activities—grocery shopping, hedge trimming, jogging, kitchen cleaning, laundry, and motorbike servicing. Each activity will be performed once, one at a time. The order in which the activities are performed is subject to the following conditions:

      Grocery shopping has to be immediately after hedge trimming.

      Kitchen cleaning has to be earlier than grocery shopping.

      Motorbike servicing has to be earlier than laundry.

      Motorbike servicing has to be either immediately before or immediately after jogging.

    Question 7

      If laundry is earlier than kitchen cleaning, then hedge trimming must be

      fifth

      fourth

      third

      second

      first

    Explanation for Question 7

      This problem is concerned with determining the order in which six activities will be performed. As with many questions involving relative ordering or ranking, it is likely that you will find it useful to diagram the various relationships given in the passage.

      The first condition in the passage tells us that grocery shopping has to be immediately after hedge trimming, which we can abbreviate as follows:

      1. HG

      The second condition tells us that kitchen cleaning has to be earlier than grocery shopping, which we can abbreviate as follows, where “...” is used to represent “earlier than” (which means any time before, including immediately before):

      2. K ... G

      The third condition tells us that motorbike servicing has to be earlier than laundry, and the fourth condition tells us that motorbike servicing has to be either immediately before or immediately after jogging. These conditions can be abbreviated as follows, where the / symbol is used to represent “or”:

      3. M ... L

      4. MJ / JM

      Notice that the information specified in these four conditions can be collapsed into two ordering statements:

      I. K ... HG (first and second conditions)

      II. MJ / JM ... L (third and fourth conditions)

      Question 7 introduces the new supposition “laundry is earlier than kitchen cleaning”:

      L ... K

      This new supposition works to further collapse the ordering statements in I and II to the single statement below; that is, if L must be earlier than K, then we know that the activities must be ordered like this:

      MJ / JM ... L ... K ... HG

      So, with the addition of the new supposition, there are exactly two possible orderings of the six activities, differing only with respect to whether motorbike servicing is immediately before or immediately after jogging:

      123456

      MJLKHG

      JMLKHG

      Question 7 asks what position hedge trimming must be in, given the new supposition. What we see here is that hedge trimming must be the fifth activity performed, and so answer choice (A) is correct.

      This was an easy question, based on the number of test takers who answered it correctly when it appeared on the LSAT. The most commonly selected incorrect answer choices were response (B) and response (C).

    Question 8

      Which one of the following, if substituted for the condition that motorbike servicing has to be earlier than laundry, would have the same effect in determining the order of the student’s activities?

      Laundry has to be one of the last three activities.

      Laundry has to be either immediately before or immediately after jogging.

      Jogging has to be earlier than laundry.

      Laundry has to be earlier than hedge trimming.

      Laundry has to be earlier than jogging.

    Explanation for Question 8

      This question asks you to select the condition which, if substituted for the third condition in the passage (repeated below), would have the same effect as the original condition.

      Third condition: Motorbike servicing has to be earlier than laundry.

      In this case, you can deduce that the correct answer choice is (C):

      (C) Jogging has to be earlier than laundry.

      The fourth condition in the passage tells you that motorbike servicing has to be either immediately before or immediately after jogging. That is, M and J must be ordered as a block, either MJ or JM, with respect to the other four activities. Thus, if, as the original third condition states, M has to be earlier than L, then we know that J must also be earlier than L. Conversely, if, as the new condition in answer choice (C) states, J has to be earlier than L, then we know that M must also be earlier than L. In short, the third condition and answer choice (C) have exactly the same effect. Therefore, answer choice (C) is correct.

      Another way to approach this kind of question is to attempt to eliminate all of the incorrect answer choices. Under this approach, you want to rule out any answer choice that does either of the following:

      rules out outcomes that the original condition allows

      allows outcomes that the original condition rules out

      Let’s see how this approach would enable us to eliminate answer choices (A), (B), (D), and (E).

      Consider the condition presented in answer choice (A):

      (A) Laundry has to be one of the last three activities.

      We can first ask whether this condition would rule out outcomes that the original third condition allows. To answer this question, we must simply determine whether there is an outcome allowed by the original third condition along with the other conditions in which laundry is one of the first three activities. Here is such an outcome:

      123456

      MJLKHG

      Because the original third condition allows this outcome, but the condition in answer choice (A) does not, answer choice (A) cannot be correct.

      Consider answer choice (B):

      (B) Laundry has to be either immediately before or immediately after jogging.

      Again, we want to first determine whether this new condition would rule out outcomes that the original third condition allows. To answer this question, we must simply determine whether there is at least one outcome allowed by the original third condition along with the other conditions in which laundry is neither immediately before nor immediately after jogging. Here is one such outcome:

      123456

      KHGJML

      This outcome, although allowed by the original third condition, would be ruled out by the alternative condition given in answer choice (B). Thus, answer choice (B) cannot be correct.

      Next consider answer choice (D):

      (D) Laundry has to be earlier than hedge trimming.

      Again, we want to first determine whether this new condition would rule out outcomes that the original third condition allows. To answer this question, we must simply determine whether there is at least one outcome allowed by the original third condition along with the other conditions in which laundry is not earlier than hedge trimming. One such outcome was given immediately above: since L is not earlier than H in this outcome, it would be ruled out by the condition in answer choice (D). So, answer choice (D) rules out an outcome that the original third condition allows, and therefore (D) cannot be the correct answer choice.

      Finally, consider answer choice (E):

      (E) Laundry has to be earlier than jogging.

      Again, we want to first determine whether having this new condition would rule out outcomes that are allowed when the original third condition is in place. To answer this question, we must simply determine whether there is at least one outcome allowed by the original third condition along with the other conditions in which laundry is not earlier than jogging. One such outcome was given above: since L is not earlier than J in this outcome, it would be ruled out by the condition presented in answer choice (E). So, answer choice (E) rules out an outcome that the original third condition allows, and therefore (E) cannot be the correct answer choice.

      In sum, answer choices (A), (B), (D), and (E) can all be eliminated because in each case the condition is one that rules out outcomes that the original condition allows. For this particular question, there was no need to consider whether any of the options could be eliminated because they allowed outcomes that the original condition ruled out.

      This question was of middle difficulty, based on the number of test takers who answered it correctly when it appeared on the LSAT. The most commonly selected incorrect answer choices were response (A) and response (B).


本文地址:https://www.ivyeducation.cn/lsattuili/7330.html

以上便是“LSAT分析推理之官网样题(五)”的全部内容,更多最新资讯请关注常春藤教育。

标签:lsat分析推理lsat备考

- 声明 -

1、由于考试政策等各方面情况的不断调整与变化,本网站所提供的考试信息仅供参考,请以权威部门公布的正式信息为准。

2、本文内容、图片由互联网用户自发贡献,该文观点仅代表作者本人,本站仅提供信息存储空间服务,不拥有所有权,不承担相关法律责任。如您发现本站有涉嫌抄袭/侵权/违法违规的内容信息,请发送邮件至shanghai60@cedca.cn举报,一经查实,本站将立即删除。